Форум » Флейм » Что такое Ri? » Ответить

Что такое Ri?

RedStar: Прошу высказать полное обоснование необходимости расчетов Ri. Сколько сталкиваюсь, столько прихожу к мнению, что знать Ri нет необходимости.

Ответов - 299, стр: 1 2 3 4 5 6 7 8 9 10 11 12 13 14 15 All

RedStar: Так зачем же "отправной точкой" используете Ri для вычислений, что резистивных, что тран-ных? Включая Альфу и примерные значения от 2 до 10 (или 0,1-0,5) Ri. Ученик пишет: в начале расчёта тебе известны точки K, P и L. Откуда ты взял, или предполагаешь брать эти данные? Известна только Рабочая точка, которую выбираем. Остальное зависит от требуемой выходной амплитуды.Если выходное сопротивление каскада - параллельная. А если, например, влияние пульсаций анодного источника - последовательная. Это как? В транс-ном - последовательная? Нет?

Кузьмич: RedStar пишет: (или 0,1-0,5) Ri А это что? И откуда взято???Это как? В транс-ном - последовательная? Нет? По переменному току верхний "конец" по схеме анодного резистора, или верхний "конец" обмотки трансформатора "сидит" на общем проводе через конденсатор большой ёмкости БП, т.е. параллельно лампе, т.е. её внутреннему сопротивлению Ri. А этот конденсатор большой ёмкости БП на НЧ( не говоря уже для более высоких частот) имеет сопротивление близкое к нулю.

RedStar: Кузьмич пишет:Дожили...Покажите пример. (не ради забавы). То, что: Кузьмич пишет: По переменному току верхний "конец" по схеме анодного резистора, или верхний "конец" обмотки трансформатора "сидит" на общем проводе через конденсатор большой ёмкости БП, Естественно. А между транс-ром (нижний вывод тр-ра) и лампой (анодом) есть что либо? Считаете это параллельным? Добавлю: "...ток пойдет по цепи из последовательно соединенного га с параллельной ветвью RL и LP ." http://www.vestnikara.spb.ru/vestn/n3/moir.htm


Кузьмич: RedStar пишет:Покажите пример. (не ради забавы). Цыкин, раздел 5,3 Трансформаторный каскад. Стр. 132 и далее.А между транс-ром (нижний вывод тр-ра) и лампой (анодом) есть что либо? Есть! Твоя любимая амплитуда, которую надо втиснуть куда-то далее. Ещё раз спрашиваю: Откуда это взято: RedStar пишет: (или 0,1-0,5) Ri.???

Ученик: RedStar пишет: Так зачем же "отправной точкой" используете Ri для вычислений Кто использует????Известна только Рабочая точка, которую выбираем. Остальное зависит от требуемой выходной амплитуды. Рабочая точка - это одна точка. У тебя - три. Объясни, откуда ты берёшь точки K и L.Это как? В транс-ном - последовательная? Хоть в каком. Выше написал: если рассматриваешь влияние пульсаций анодного источника, по простому - фон от этих пульсаций, то здесь рассматривается как обычный делитель: Rа включена с Ri последовательно.

Пермяк: В начале этой темы RedStar писал:До сих пор не пойму реальное применение, хотя прекрасно знаю, что представляет из себя внутреннее.зачем нужно Ri. Никто не отвечает по сути, полным объемом.1. Ri драйверной лампы нужно знать, чтобы вычислить выходное сопротивление драйвера Rвых. Если каскад резистивный, то его выходное сопротивление равно: Rвых=Ra||Ri Если Rвых составляет 10% и более от величины гридлика выходной лампы, то разделительный конденсатор Ср рассчитывают с учётом Rвых драйвера. При наличии в драйверном каскаде ООС (например, Rк не зашунтирован ёмкостью), то Ri.св лампы возрастёт: , и его тоже надо узнать (вычислить), чтобы определить величину Ср. Некто может возразить, что Ср легче подобрать, чем рассчитывать, но, во-первых, для этого нужен запас разных по ёмкости конденсаторов, а во-вторых, зачем тогда нужна векторная алгебра, если детали подбираем "методом тыка"? 2. Ri лампы в выходном каскаде также участвует в формировании выходного сопротивления: Чем меньше Rвых, тем лучше коэффициент демпфирования подвижной системы динамика. Вывод: Ri - важный параметр лампы, и его, так же, как как µ и S желательно знать.

Aleph: RedStar пишет:Так зачем же такой "отправной точкой" используете Ri для вычислений, что резистивных, что трансформаторных? Включая Альфу и примерные значения от 2 до 10 (или 0,1-0,5)Ri. Да никто из умеющих проводить расчёт не использует эти соотношения, они просто показывают, в каких примерно пределах может находиться значение Ri, для новичков, просто для сведения. Кто умеет - строит нагрузку по ВАХ.

RedStar: Ученик пишет: Объясни, откуда ты берёшь точки K и L.Зачем? Вы же все считаете, что у меня всё идет вразрез физике и закону дедушки Ома. Кроме РТ есть еще два значения, - питание - Ea, и требуемая выходная амплитуда. От них надо исходить. Aleph пишет: никто из умеющих проводить расчёт не использует эти соотношения, они просто показывают,... Далее и без Ri можно обойтись, верно? Уже говорил, что даже по примерным ВАХ можно точно настроить каскад. Ведь только формулами можно доказать выходное сопротивление. Или нет?

aleks8845: RedStar пишет: Далее и без Ri можно обойтись, верно? Если можно так выразитmся, пусть меня поправят, при определении РТ пусть Ri не будет являться? самым главным расчетным параметром , но в дальнейшем ,оно (Ri), все равно, будет иметь значение. В резистивном каскаде будет влиять на выходное сопротивление каскада, а в трансформаторном.... К примеру мне нужен трансформаторный драйвер, я буду выбирать к МКТ низкоомную лампу, т.е. с бОлее низким Ri, и уж точно не поставлю ничего в пентодно, тетродном включении, где Ri бывает до сотни кОм...

Пермяк: RedStar пишет: и без Ri можно обойтись, верно? Моим постом выше я показал, для чего нужно ЗНАТЬ величину Ri. RedStar пишет: Ведь только формулами можно доказать выходное сопротивление. Или нет? Представьте , что нагрузка немного изменилась. например, Rдинамика уменьшилось на НЧ. Уменьшится и напряжение на динамике. И изменится оно тем сильнее, чем выше значение Rвых, которое зависит от Ri выходной лампы. Формулу я показал, поэтому вот эта Ваша Фраза свидетельствует, что Вы не не поняли:Ведь только формулами можно доказать выходное сопротивление. Или нет? Повторюсь: падение напряжения при снижении Rнагрузки показывает, что Ri - реальная физическая величина.

Ученик: RedStar пишет: Зачем? Затем. Ты ведёшь расчёт от этих точек. Так объясни, откуда ты их берёшь. RedStar пишет: Кроме РТ есть еще два значения, - питание - Ea Ты не выдумывай на ходу, Еа ты получаешь далее из своего расчёта. А расчёт ведёшь от трёх точек. Про точку Р понятно, откуда взялись K и L?

RedStar: А для случая с пентодом, у которого 30 кОм без МОС? Какое выходное сопротивление? Около 60-100 Ом? Почему должно измениться Ri при изменении нагрузки? Изменяется ведь Ra при неизменном Ктр. Что участвует как Ra? (пункт 2 из поста Леонида). W1 = Ra? Тогда должно быть последовательно. Ученик пишет:откуда взялись K и L? Из графика, циркулем от РТ.

Ученик: RedStar пишет: Из графика, циркулем от РТ.1 Из какого "графика"? Дежурств по кухне? 2 Циркулем - это будет окружность. 3 Почему ты поставил точки в этих местах окружности - непонятно. P.S. Ты сочинил какой-то расчёт. Что является целью расчёта, откуда и как берёшь исходные данные, почему считаешь, что так будет лучше (в сравнении с чем?)...никакой информации. Почему из тебя всё клещами надо вытаскивать?

RedStar: 1. А разве не понятно, что ВАХ. 2. У вас циркулем только окружность? Про измерения им расстояний никак не догадались? Сказал ведь, что хватит про мой расчет. Ходите вокруг да около не понимая.

Пермяк: RedStar пишет: А для случая с пентодом, у которого 30 кОм без МОС? Какое выходное сопротивление? Около 60-100 Ом? Я Вам писал выше: Rвых=Ra||Ri, т.е. параллельному соединению внутреннего и анодного сопротивлений. Умеете считать сопро двух параллельно соединённых?

Ученик: RedStar пишет: А разве не понятно, что ВАХ. Так-так...У вас циркулем только окружность? Циркуль для того и придуман. А ты предлагаешь тыкать циркулем в экран монитора, или ВАХ в книге, а потом прикладывать его к линейке? Офигенная "точность", и удобство.Сказал ведь, что хватит про мой расчет. Нет, не хватит. В сухом остатке: тебя чем-то не устраивала точность графического метода. Поскрипев мозгами, ты взял те же самые графические ВАХ, но выбрал для отсчёта не крайние точки, а близлежащие к Рт. То, что точность от этого может только пострадать, в голову тебе не пришло. Ну ладно, взял и взял. Но потом проводишь какие-то идиотские громоздкие подсчёты...а просто считать с осей значения сложно? Вот же, на твоём графике Ua0 = 270 Iam = 13,5. Или же составить обычные пропорции, как в начальной школе учили - ума не хватило? Полный капец...векторная алгебра. Наткнулся:

RedStar: Ученик пишет: Циркуль для того и придуман. О как. Ци́ркуль — инструмент для черчения окружностей и дуг, также может быть использован для измерения расстояний, в частности, на картах. Причем здесь монитор? Есть принтер, распечатать, а потом проводить измерения.точность от этого может только пострадать Больше страдает точность от 2-10Ri, или в пентоде 0.2-0.5Ri. (заодно ответил на вопрос Кузьмича).Вот же, на твоём графике Ua0 = 270, Iam = 13,5. Маленько ошиблись: Ea=270, Imax=13.5. А точки Ua0 = 190, Iam = 1,0Наткнулся:Ой, тоже мне, нашли столетней давности, взятое из одной хорошей книги. И что?

Aleph: RedStar, Вы бы заявку на нобелевку дали, что ли (или может на премию Дарвина?). Почти 100 лет все считают как в учебнике, а тут Вы новый метод открыли. Да ещё и всех тайн методики не пишете, только обзываете всех неучами. Вы рискните на Вегалабе написать. Там быстро бан влепят за "тайные знания, недоступные простым физикам". Пермяк же написал доступно, хоть глотай и переваривай:Представьте , что нагрузка немного изменилась, например, Rдинамика уменьшилось на НЧ. Уменьшится и напряжение на динамике. И изменится оно тем сильнее, чем выше значение Rвых, которое зависит от Ri выходной лампы. Повторюсь: падение напряжения при снижении Rнагрузки показывает, что Ri - реальная физическая величина.

Ученик: RedStar пишет:Больше страдает точность от 2-10Ri, или в пентоде 0.2-0.5Ri. Вам уже объясняли, и не раз, что в большинстве случаев никто не считал и не считает нагрузку строго от Ri. Типа нагрузка д.б. = 4,15Ri. Иногда задаются альфой не меньше какой-то определённой величины. А какой конкретно - выбирает разработчик, хочет мощность побольше - одно, хочет высокий коэффициент демпфирования - другое. Для пентодов в звуковых цепях Ri вообще не учитывается. Сколько раз повторять?Маленько ошиблись: Ea=270, Imax=13.5. Видишь, ты понял о чём речь, ибо я не поленился, и пояснил про точки пересечения нагрузочной прямой с осями. Что же мешает тебе (уж не меньше года, наверное) сосредоточиться, и вместо обрывков-ребусов оформить свои мысли в нормальную форму? Ну так что там про точность "твоего метода"?Ой, тоже мне, нашли столетней давности, взятое из одной хорошей книги. Да из какой книги? Такая чушь только в твоей голове родиться могла. Aleph пишет: а тут Вы новый метод открыли. Да какой там новый метод. Вбил себе в голову, что нужна какая-то суперточность в этих вопросах... вот и пыжится. Сколько раз писали, что всё это усреднённое, лампы отличаются, да и нет существенных изменений от вариаций +/- 5, а часто и 10% - хоть кол на голове теши. Это называется просто - навязчивая мысль.

Пермяк: RedStar пишет: Ведь только формулами можно доказать выходное сопротивление. Или нет? Выходное сопротивление можно измерить. Причём, разными методами. Например, для трансформаторного каскада, "метод двух нагрузок": Rвых резистивного каскада можно измерить таким же методом. Нагрузкой резистивного каскада является сеточный резистор Rg последующего каскада, для драйвера - выходного. Подаём на вход каскада сигнал такой величины, чтобы переменка на Rg стала равной, скажем, U1=5В. Затем, параллельно Rg подключаем резистор R2 такой величины, чтобы напряжение на нём уменьшилось, скажем, до U2=4В. Формула для вычисления Rвых будет такой: В формуле R1 - это Rg.



полная версия страницы